+1 Daumen
9,3k Aufrufe

Hi leute,

ich hab erneut Probleme und brauch eure Hilfe. Diesmal gehts um die Diracsche Deltafunktion.

Wir sollen ausgehend von

$$\int _{ -\infty  }^{ \infty  }{ f(x)\delta (x)dx\quad =\quad f(0) } $$

und der Heaviside-Funktion

$$\int _{ -\infty  }^{ \infty  }{ f(x)\theta (x)dx\quad =\quad \int _{ 0 }^{ \infty  }{ f(x)dx }  }$$

die Folgenen Eigenschaften zeigen:

1) $$\int _{ -\infty  }^{ \infty  }{ f(x)\delta (x-a)dx\quad =\quad f(a) } $$

2) $$\delta (-x)=\delta (x)$$

und

$$\delta (ax)=\frac { 1 }{ |\alpha | } \delta (x)$$ mit alpha in R

3) $$\int _{ -\infty  }^{ \infty  }{ g(x)\delta (f(x))\quad =\quad \sum _{ i }^{  }{ g({ x }_{ i })\frac { 1 }{ |f'({ x }_{ i })| }  }  } $$ wobei die Summe über die Nullstellen xi von f geht

4) $$\frac { d }{ dx } \theta (x)\quad =\quad \delta \quad (x)$$

Ich bin einfach überfragt (und vielleicht so zum ende hin auch etwas überarbeitet) und bin wirklich dankbar für jede Hilfe.

Liebe Grüße

EDIT: 2) korrigiert.

Avatar von

3 Antworten

+1 Daumen
 
Beste Antwort

Der Hinweis bezieht sich darauf, dass es ueberhaupt keine Funktion \(\delta(x)\) geben kann, die die "definierende" Eigenschaft \(\int f(x)\delta(x)\,dx=f(0)\) haette. Das sollst Du einfach ignorieren und munter so rechnen, als gaebe es ueberhaupt kein Problem. Ein Beispiel zur Inspiration: Mit der Substitution \(y=-x\) hast Du $$\int f(x)\delta(-x)\,dx=\int f(-y)\delta(y)\,dy=f(0)=\int f(x)\delta(x)\,dx.$$ Daraus siehst Du dass \(\delta(-x)\) die gleiche Wirkung im Integral hat wie \(\delta(x)\). Daraus folgt für den Physiker \(\delta(-x)=\delta(x)\). Der Rest nach demselben Motto. Viel Spass.

Avatar von

Du substituierst y = -x, und hast dann dy/dx = - 1 . Und damit dy = - dx. D.h. du solltest dx mit -dy ersetzen bei der Substitution. Gleichzeitig ist nun aber die untere Grenze minus unendlich und die obere Grenze ist plus unendlich. Wenn man die Grenzen vertauscht, ist das vorher vermisste Minus wieder weg. Daher stimmt schon mal das erste "gleich" deiner Umformung.

+2 Daumen

Du machst es dir etwas zu schwer ;)

$$ \int_{-\infty}^{\infty}f(x)\delta(x-a)dx\\u=x-a,,x=u+a,dx=du\\\int_{-\infty}^{\infty}f(x)\delta(x-a)dx=\int_{-\infty}^{\infty}f(u+a)\delta(u)du=f(0+a)=f(a) $$

Avatar von 37 k

Merke schon das ich es mir zu schwer mache, danke euch beiden. Eine frage jedoch: warum genau ist das Integral von f (u+a) = f (0+a). Mir is klar das u integriert mit den Grenzen 0 ist. Aber muss a nicht auch integriert werden. Es ist doch eigentlich eine konstante....oder nich?


Und dann gehts weiter mit 2). Da hatte ja fakename zum ersten Teil ja schon einen guten Anzatz gegeben (glaub ich). Aber beim zweiten Teil bin ich mir nicht sicher was ich machen könnte.

Beim zweiten Teil in 2) substituierst Du wieder \(y=ax\), wobei Du die Faelle \(a>0\) und \(a<0\) unterscheidest.

Bei 3) kannst Du Dir ueberlegen, dass \(\delta\) nur da wirkt, wo \(f(x)=0\) wird, und dass es reicht, ueber beliebig kleine Intervalle um die Nullstellen herum zu integrieren: $$\int_{-\infty}^\infty g(x)\delta(f(x))\,dx=\sum_i \int_{x_i-\epsilon}^{x_i+\epsilon}g(x)\delta(f(x))\,dx.$$ Ueberlege Dir dann, wie man den zweiten Teil von 2) hier benutzen kann. (Linearisiere, Du Physiker!)

Bei 4) kannst Du $$\int_{-a}^af(x)\theta'(x)\,dx$$ partiell integrieren, um zu sehen, dass \(\theta'(x)\) im Integral genauso wirkt wie \(\delta(x)\).

So. Jetzt bist Du dran. Die Punkte wollen verdient sein!

+1 Daumen

Hast du bei 1) eine Substitution mit u = x-a versucht?

D.h. dann dx = du und x = u+a

Avatar von 162 k 🚀

Du studierst Physik oder so, nicht Mathematik, oder?

Genau!

Lu ich werde den Ansatz mit der substituierung später mal ausprobiern

Dann stell Dich einfach bloed. :) Tu so, als sei δ eine ganz normale Funktion und benutze die ueblichen Rechenregeln für Integrale, insbesondere Substitution. Das wird auch so erwartet.

Toll jetzt werd ich nicht nur in den Mathe Vorlesungen als Physiker anders behandelt (scherz) ;3

Also spricht ich lasse einfach (x) weg? Aber wie wäre es dann bei der Aufgabe 3) mit δ(f(x) (und ja, die Woche war lang und ich stell mich etwas dusselig an >.<)

Ich danke dir schonmal. Ich weiß nicht ob ich einfach voll auf dem schlauch stehe aber ich werd langsam bekloppt. (Zusätzlich zu dem dass ich daran schon so lange sitze brauch ich die Punkte TT)

Ich hab das jetzt so verstanden, dass ich einfach mal rechne. Spricht das integral von f (x) und delta (x-a). Das hab ich mit der sub. X=u-a. Ich komm einfach auf keine sinnvolle Lösung. Ich hab dann sowas wie

(Sorry bin am Handy)

(X+a^2+ax^2)/(2) mit den Grenzen -unend.. Bis unend.

Wo is mein Fehler?

(Sorry mein handy will es einfach nich richtig rum einfügen -,-)

Bild Mathematik

nach dem "du" kannst du direkt die definierende Eigenschaft für delta benutzen und "= f(0+a) = f(a) schreiben.

Schreibe vielleicht erst mal die Voraussetzung a) mit andern Buchstaben, damit du nicht durcheinanderkommst mit zu vielen gleichen Buchstaben.

$$\int _{ -\infty  }^{ \infty  }{ f(x)\delta (x)dx\quad =\quad f(0) } $$

$$\int _{ -\infty  }^{ \infty  }{ g(u)\delta (u)du\quad =\quad g(0) } $$

Danach kannst du einfacher mit dem f in der Behauptung umgehen.

 g(u) = f(u+a) und g(0) = f(0+a)

EDIT: Habe jetzt die Antworten etwas auseinandergenommen, weil es etwas mühsam wurde, allen Baustellen gleichzeitig zu folgen. Ging aber nur beschränkt.

Die Antwort von jc2144 ist jetzt die ganze Rechnung zu (1) . Mit meinem Kommentar gerade eben solltest du seine Rechnung nachvollziehen können.

Achtung, was jetzt folgt, ist weder mathematisch noch physikalisch wasserdicht. Ein Veranschauungsversuch.

Die Diracsche Deltafunktion d wird auch Diracstoss genannt. Sie ist so definiert, dass sie bestimmte Funktionswerte aus dem andern Faktor herauspflücken kann. delta(0) ist quasi so stark unendlich, dass es nichts ausmacht, dass das Intervall in dem delta(x) nicht 0 ist, die Breite 0 hat.  

Die Heavisidefunktion kann benutzt werden, nur über einen gewissen Bereich des beidseits unendlichen Intervalls zu integrieren. Sie ist auf einer Seite der y-Achse 0 und auf der andern 1. 

Vgl. z.B. auch hier: https://www.mathelounge.de/328867/ausblendeigenschaft-des-dirac-impulses 

Ein anderes Problem?

Stell deine Frage

Willkommen bei der Mathelounge! Stell deine Frage einfach und kostenlos

x
Made by a lovely community